Determining the length of a curve using integrals [closed]












0












$begingroup$


Im having problems solving the following problem.



Determine the distance of the following curve:
begin{cases}
x(t)= cos^3(t),\
y(t)= sin^3(t). \
end{cases}

In this case $0 le t le 2pi$



The answer should be $6$



I would be really grateful if someone could of help me understand it.










share|cite|improve this question











$endgroup$



closed as off-topic by Namaste, Eevee Trainer, Paul Frost, KReiser, Lord_Farin Dec 26 '18 at 9:37


This question appears to be off-topic. The users who voted to close gave this specific reason:


  • "This question is missing context or other details: Please provide additional context, which ideally explains why the question is relevant to you and our community. Some forms of context include: background and motivation, relevant definitions, source, possible strategies, your current progress, why the question is interesting or important, etc." – Namaste, Eevee Trainer, Paul Frost, KReiser, Lord_Farin

If this question can be reworded to fit the rules in the help center, please edit the question.





















    0












    $begingroup$


    Im having problems solving the following problem.



    Determine the distance of the following curve:
    begin{cases}
    x(t)= cos^3(t),\
    y(t)= sin^3(t). \
    end{cases}

    In this case $0 le t le 2pi$



    The answer should be $6$



    I would be really grateful if someone could of help me understand it.










    share|cite|improve this question











    $endgroup$



    closed as off-topic by Namaste, Eevee Trainer, Paul Frost, KReiser, Lord_Farin Dec 26 '18 at 9:37


    This question appears to be off-topic. The users who voted to close gave this specific reason:


    • "This question is missing context or other details: Please provide additional context, which ideally explains why the question is relevant to you and our community. Some forms of context include: background and motivation, relevant definitions, source, possible strategies, your current progress, why the question is interesting or important, etc." – Namaste, Eevee Trainer, Paul Frost, KReiser, Lord_Farin

    If this question can be reworded to fit the rules in the help center, please edit the question.



















      0












      0








      0


      0



      $begingroup$


      Im having problems solving the following problem.



      Determine the distance of the following curve:
      begin{cases}
      x(t)= cos^3(t),\
      y(t)= sin^3(t). \
      end{cases}

      In this case $0 le t le 2pi$



      The answer should be $6$



      I would be really grateful if someone could of help me understand it.










      share|cite|improve this question











      $endgroup$




      Im having problems solving the following problem.



      Determine the distance of the following curve:
      begin{cases}
      x(t)= cos^3(t),\
      y(t)= sin^3(t). \
      end{cases}

      In this case $0 le t le 2pi$



      The answer should be $6$



      I would be really grateful if someone could of help me understand it.







      integration






      share|cite|improve this question















      share|cite|improve this question













      share|cite|improve this question




      share|cite|improve this question








      edited Dec 26 '18 at 7:28









      Dylan

      14.3k31127




      14.3k31127










      asked Dec 25 '18 at 22:03









      FosorfFosorf

      63




      63




      closed as off-topic by Namaste, Eevee Trainer, Paul Frost, KReiser, Lord_Farin Dec 26 '18 at 9:37


      This question appears to be off-topic. The users who voted to close gave this specific reason:


      • "This question is missing context or other details: Please provide additional context, which ideally explains why the question is relevant to you and our community. Some forms of context include: background and motivation, relevant definitions, source, possible strategies, your current progress, why the question is interesting or important, etc." – Namaste, Eevee Trainer, Paul Frost, KReiser, Lord_Farin

      If this question can be reworded to fit the rules in the help center, please edit the question.







      closed as off-topic by Namaste, Eevee Trainer, Paul Frost, KReiser, Lord_Farin Dec 26 '18 at 9:37


      This question appears to be off-topic. The users who voted to close gave this specific reason:


      • "This question is missing context or other details: Please provide additional context, which ideally explains why the question is relevant to you and our community. Some forms of context include: background and motivation, relevant definitions, source, possible strategies, your current progress, why the question is interesting or important, etc." – Namaste, Eevee Trainer, Paul Frost, KReiser, Lord_Farin

      If this question can be reworded to fit the rules in the help center, please edit the question.






















          2 Answers
          2






          active

          oldest

          votes


















          3












          $begingroup$

          The length of the curve, known as arc length, is given by
          $$L = int_{0}^{2pi}sqrt{x'(t)^{2}+y'(t)^{2}} dt$$
          In this case, this gives
          begin{align}
          L&=int_{0}^{2pi}sqrt{(3cos^{2}{t}(-sin{t}))^{2}+(3sin^{2}{t}(cos{t}))^{2}} dt \
          &= int_{0}^{2pi}sqrt{9cos^{4}{t}sin^{2}{t}+9sin^{4}{t}cos^{2}{t}} dt\
          &= int_{0}^{2pi}sqrt{9cos^{2}{t}sin^{2}{t}(sin^{2}{t}+cos^{2}{t})} dt\
          &= int_{0}^{2pi}3|cos{t}sin{t}| dt\
          &= int_{0}^{2pi}frac{3}{2}|sin{2t}| dt
          end{align}

          You can easily verify that $sin{2t}
          geq 0$
          when $tin[0,frac{pi}{2}]cup[pi,frac{3pi}{2}]$ and that $sin{2t}leq0$ when $tin[frac{pi}{2},pi]cup[frac{3pi}{2},2pi].$ Therefore
          $$L=frac{3}{2}left[int_{0}^{pi/2}sin{2t} dt + int_{pi}^{3pi/2}sin{2t} dt - int_{pi/2}^{pi}sin{2t} dt - int_{3pi/2}^{2pi}sin{2t} dt right]$$
          You can also check that the first two integrals are $1$ and the last two are $-1$. Therefore
          $$L=frac{3}{2}(4)=6$$






          share|cite|improve this answer









          $endgroup$





















            3












            $begingroup$

            If you don't know the formula imagine a little particle moving along this path. Then at each small slice of time, the particle moves its total speed times time. That is, if the particle's path is described a vector-valued function of time $$overrightarrow{r}(t) = left langle begin{matrix} cos ^3 (t) \ sin^3 (t)end{matrix} right rangle $$ then we have that $$ text{d}s = | r'(t) | text{d}t $$ so that $$s = int_{t_1}^{t_2} sqrt{(x'(t))^2 + (y'(t))^2} text{d}t$$ From here on, the solution is identical to pwerth's excellent derivation, so in your case the arc length is $$ s= int_0^{2pi}sqrt{left (frac{text{d}}{text{d}x} cos^3 (t) right)^2 + left (frac{text{d}}{text{d}x} sin^3 (t) right)^2 } text{d}t $$ $$= int_0^{2pi} sqrt{left(3 cos^2(t) cdot (- sin(t)) right)^2 + left(3 sin^2(t) cos(t) right)^2} text{d}t $$ $$= int_0^{2pi} sqrt{9 cos^4(t)sin^2(t) + 9 sin^4(t) cos^2(t)} text{d}t $$ $$=int_0^{2pi} |3sin(t) cos(t)| sqrt{sin^2(t) + cos^2(t)} text{d}t $$ $$ = 3 int_0^{2pi} frac{1}{2} |sin(2t)| text{d}t = frac{3}{2} cdot 4 = 6 $$






            share|cite|improve this answer









            $endgroup$




















              2 Answers
              2






              active

              oldest

              votes








              2 Answers
              2






              active

              oldest

              votes









              active

              oldest

              votes






              active

              oldest

              votes









              3












              $begingroup$

              The length of the curve, known as arc length, is given by
              $$L = int_{0}^{2pi}sqrt{x'(t)^{2}+y'(t)^{2}} dt$$
              In this case, this gives
              begin{align}
              L&=int_{0}^{2pi}sqrt{(3cos^{2}{t}(-sin{t}))^{2}+(3sin^{2}{t}(cos{t}))^{2}} dt \
              &= int_{0}^{2pi}sqrt{9cos^{4}{t}sin^{2}{t}+9sin^{4}{t}cos^{2}{t}} dt\
              &= int_{0}^{2pi}sqrt{9cos^{2}{t}sin^{2}{t}(sin^{2}{t}+cos^{2}{t})} dt\
              &= int_{0}^{2pi}3|cos{t}sin{t}| dt\
              &= int_{0}^{2pi}frac{3}{2}|sin{2t}| dt
              end{align}

              You can easily verify that $sin{2t}
              geq 0$
              when $tin[0,frac{pi}{2}]cup[pi,frac{3pi}{2}]$ and that $sin{2t}leq0$ when $tin[frac{pi}{2},pi]cup[frac{3pi}{2},2pi].$ Therefore
              $$L=frac{3}{2}left[int_{0}^{pi/2}sin{2t} dt + int_{pi}^{3pi/2}sin{2t} dt - int_{pi/2}^{pi}sin{2t} dt - int_{3pi/2}^{2pi}sin{2t} dt right]$$
              You can also check that the first two integrals are $1$ and the last two are $-1$. Therefore
              $$L=frac{3}{2}(4)=6$$






              share|cite|improve this answer









              $endgroup$


















                3












                $begingroup$

                The length of the curve, known as arc length, is given by
                $$L = int_{0}^{2pi}sqrt{x'(t)^{2}+y'(t)^{2}} dt$$
                In this case, this gives
                begin{align}
                L&=int_{0}^{2pi}sqrt{(3cos^{2}{t}(-sin{t}))^{2}+(3sin^{2}{t}(cos{t}))^{2}} dt \
                &= int_{0}^{2pi}sqrt{9cos^{4}{t}sin^{2}{t}+9sin^{4}{t}cos^{2}{t}} dt\
                &= int_{0}^{2pi}sqrt{9cos^{2}{t}sin^{2}{t}(sin^{2}{t}+cos^{2}{t})} dt\
                &= int_{0}^{2pi}3|cos{t}sin{t}| dt\
                &= int_{0}^{2pi}frac{3}{2}|sin{2t}| dt
                end{align}

                You can easily verify that $sin{2t}
                geq 0$
                when $tin[0,frac{pi}{2}]cup[pi,frac{3pi}{2}]$ and that $sin{2t}leq0$ when $tin[frac{pi}{2},pi]cup[frac{3pi}{2},2pi].$ Therefore
                $$L=frac{3}{2}left[int_{0}^{pi/2}sin{2t} dt + int_{pi}^{3pi/2}sin{2t} dt - int_{pi/2}^{pi}sin{2t} dt - int_{3pi/2}^{2pi}sin{2t} dt right]$$
                You can also check that the first two integrals are $1$ and the last two are $-1$. Therefore
                $$L=frac{3}{2}(4)=6$$






                share|cite|improve this answer









                $endgroup$
















                  3












                  3








                  3





                  $begingroup$

                  The length of the curve, known as arc length, is given by
                  $$L = int_{0}^{2pi}sqrt{x'(t)^{2}+y'(t)^{2}} dt$$
                  In this case, this gives
                  begin{align}
                  L&=int_{0}^{2pi}sqrt{(3cos^{2}{t}(-sin{t}))^{2}+(3sin^{2}{t}(cos{t}))^{2}} dt \
                  &= int_{0}^{2pi}sqrt{9cos^{4}{t}sin^{2}{t}+9sin^{4}{t}cos^{2}{t}} dt\
                  &= int_{0}^{2pi}sqrt{9cos^{2}{t}sin^{2}{t}(sin^{2}{t}+cos^{2}{t})} dt\
                  &= int_{0}^{2pi}3|cos{t}sin{t}| dt\
                  &= int_{0}^{2pi}frac{3}{2}|sin{2t}| dt
                  end{align}

                  You can easily verify that $sin{2t}
                  geq 0$
                  when $tin[0,frac{pi}{2}]cup[pi,frac{3pi}{2}]$ and that $sin{2t}leq0$ when $tin[frac{pi}{2},pi]cup[frac{3pi}{2},2pi].$ Therefore
                  $$L=frac{3}{2}left[int_{0}^{pi/2}sin{2t} dt + int_{pi}^{3pi/2}sin{2t} dt - int_{pi/2}^{pi}sin{2t} dt - int_{3pi/2}^{2pi}sin{2t} dt right]$$
                  You can also check that the first two integrals are $1$ and the last two are $-1$. Therefore
                  $$L=frac{3}{2}(4)=6$$






                  share|cite|improve this answer









                  $endgroup$



                  The length of the curve, known as arc length, is given by
                  $$L = int_{0}^{2pi}sqrt{x'(t)^{2}+y'(t)^{2}} dt$$
                  In this case, this gives
                  begin{align}
                  L&=int_{0}^{2pi}sqrt{(3cos^{2}{t}(-sin{t}))^{2}+(3sin^{2}{t}(cos{t}))^{2}} dt \
                  &= int_{0}^{2pi}sqrt{9cos^{4}{t}sin^{2}{t}+9sin^{4}{t}cos^{2}{t}} dt\
                  &= int_{0}^{2pi}sqrt{9cos^{2}{t}sin^{2}{t}(sin^{2}{t}+cos^{2}{t})} dt\
                  &= int_{0}^{2pi}3|cos{t}sin{t}| dt\
                  &= int_{0}^{2pi}frac{3}{2}|sin{2t}| dt
                  end{align}

                  You can easily verify that $sin{2t}
                  geq 0$
                  when $tin[0,frac{pi}{2}]cup[pi,frac{3pi}{2}]$ and that $sin{2t}leq0$ when $tin[frac{pi}{2},pi]cup[frac{3pi}{2},2pi].$ Therefore
                  $$L=frac{3}{2}left[int_{0}^{pi/2}sin{2t} dt + int_{pi}^{3pi/2}sin{2t} dt - int_{pi/2}^{pi}sin{2t} dt - int_{3pi/2}^{2pi}sin{2t} dt right]$$
                  You can also check that the first two integrals are $1$ and the last two are $-1$. Therefore
                  $$L=frac{3}{2}(4)=6$$







                  share|cite|improve this answer












                  share|cite|improve this answer



                  share|cite|improve this answer










                  answered Dec 25 '18 at 22:22









                  pwerthpwerth

                  3,340417




                  3,340417























                      3












                      $begingroup$

                      If you don't know the formula imagine a little particle moving along this path. Then at each small slice of time, the particle moves its total speed times time. That is, if the particle's path is described a vector-valued function of time $$overrightarrow{r}(t) = left langle begin{matrix} cos ^3 (t) \ sin^3 (t)end{matrix} right rangle $$ then we have that $$ text{d}s = | r'(t) | text{d}t $$ so that $$s = int_{t_1}^{t_2} sqrt{(x'(t))^2 + (y'(t))^2} text{d}t$$ From here on, the solution is identical to pwerth's excellent derivation, so in your case the arc length is $$ s= int_0^{2pi}sqrt{left (frac{text{d}}{text{d}x} cos^3 (t) right)^2 + left (frac{text{d}}{text{d}x} sin^3 (t) right)^2 } text{d}t $$ $$= int_0^{2pi} sqrt{left(3 cos^2(t) cdot (- sin(t)) right)^2 + left(3 sin^2(t) cos(t) right)^2} text{d}t $$ $$= int_0^{2pi} sqrt{9 cos^4(t)sin^2(t) + 9 sin^4(t) cos^2(t)} text{d}t $$ $$=int_0^{2pi} |3sin(t) cos(t)| sqrt{sin^2(t) + cos^2(t)} text{d}t $$ $$ = 3 int_0^{2pi} frac{1}{2} |sin(2t)| text{d}t = frac{3}{2} cdot 4 = 6 $$






                      share|cite|improve this answer









                      $endgroup$


















                        3












                        $begingroup$

                        If you don't know the formula imagine a little particle moving along this path. Then at each small slice of time, the particle moves its total speed times time. That is, if the particle's path is described a vector-valued function of time $$overrightarrow{r}(t) = left langle begin{matrix} cos ^3 (t) \ sin^3 (t)end{matrix} right rangle $$ then we have that $$ text{d}s = | r'(t) | text{d}t $$ so that $$s = int_{t_1}^{t_2} sqrt{(x'(t))^2 + (y'(t))^2} text{d}t$$ From here on, the solution is identical to pwerth's excellent derivation, so in your case the arc length is $$ s= int_0^{2pi}sqrt{left (frac{text{d}}{text{d}x} cos^3 (t) right)^2 + left (frac{text{d}}{text{d}x} sin^3 (t) right)^2 } text{d}t $$ $$= int_0^{2pi} sqrt{left(3 cos^2(t) cdot (- sin(t)) right)^2 + left(3 sin^2(t) cos(t) right)^2} text{d}t $$ $$= int_0^{2pi} sqrt{9 cos^4(t)sin^2(t) + 9 sin^4(t) cos^2(t)} text{d}t $$ $$=int_0^{2pi} |3sin(t) cos(t)| sqrt{sin^2(t) + cos^2(t)} text{d}t $$ $$ = 3 int_0^{2pi} frac{1}{2} |sin(2t)| text{d}t = frac{3}{2} cdot 4 = 6 $$






                        share|cite|improve this answer









                        $endgroup$
















                          3












                          3








                          3





                          $begingroup$

                          If you don't know the formula imagine a little particle moving along this path. Then at each small slice of time, the particle moves its total speed times time. That is, if the particle's path is described a vector-valued function of time $$overrightarrow{r}(t) = left langle begin{matrix} cos ^3 (t) \ sin^3 (t)end{matrix} right rangle $$ then we have that $$ text{d}s = | r'(t) | text{d}t $$ so that $$s = int_{t_1}^{t_2} sqrt{(x'(t))^2 + (y'(t))^2} text{d}t$$ From here on, the solution is identical to pwerth's excellent derivation, so in your case the arc length is $$ s= int_0^{2pi}sqrt{left (frac{text{d}}{text{d}x} cos^3 (t) right)^2 + left (frac{text{d}}{text{d}x} sin^3 (t) right)^2 } text{d}t $$ $$= int_0^{2pi} sqrt{left(3 cos^2(t) cdot (- sin(t)) right)^2 + left(3 sin^2(t) cos(t) right)^2} text{d}t $$ $$= int_0^{2pi} sqrt{9 cos^4(t)sin^2(t) + 9 sin^4(t) cos^2(t)} text{d}t $$ $$=int_0^{2pi} |3sin(t) cos(t)| sqrt{sin^2(t) + cos^2(t)} text{d}t $$ $$ = 3 int_0^{2pi} frac{1}{2} |sin(2t)| text{d}t = frac{3}{2} cdot 4 = 6 $$






                          share|cite|improve this answer









                          $endgroup$



                          If you don't know the formula imagine a little particle moving along this path. Then at each small slice of time, the particle moves its total speed times time. That is, if the particle's path is described a vector-valued function of time $$overrightarrow{r}(t) = left langle begin{matrix} cos ^3 (t) \ sin^3 (t)end{matrix} right rangle $$ then we have that $$ text{d}s = | r'(t) | text{d}t $$ so that $$s = int_{t_1}^{t_2} sqrt{(x'(t))^2 + (y'(t))^2} text{d}t$$ From here on, the solution is identical to pwerth's excellent derivation, so in your case the arc length is $$ s= int_0^{2pi}sqrt{left (frac{text{d}}{text{d}x} cos^3 (t) right)^2 + left (frac{text{d}}{text{d}x} sin^3 (t) right)^2 } text{d}t $$ $$= int_0^{2pi} sqrt{left(3 cos^2(t) cdot (- sin(t)) right)^2 + left(3 sin^2(t) cos(t) right)^2} text{d}t $$ $$= int_0^{2pi} sqrt{9 cos^4(t)sin^2(t) + 9 sin^4(t) cos^2(t)} text{d}t $$ $$=int_0^{2pi} |3sin(t) cos(t)| sqrt{sin^2(t) + cos^2(t)} text{d}t $$ $$ = 3 int_0^{2pi} frac{1}{2} |sin(2t)| text{d}t = frac{3}{2} cdot 4 = 6 $$







                          share|cite|improve this answer












                          share|cite|improve this answer



                          share|cite|improve this answer










                          answered Dec 25 '18 at 22:27









                          Cade ReinbergerCade Reinberger

                          38317




                          38317















                              Popular posts from this blog

                              Le Mesnil-Réaume

                              Ida-Boy-Ed-Garten

                              web3.py web3.isConnected() returns false always